0 Daumen
301 Aufrufe

Aufgabe:

\( \sum \limits_{k=2}^{\infty} \frac{1}{(\ln k)^{k}} \) 


Problem/Ansatz:

Capture.JPG

Könnte mir jemanden bitte erklären, wie kann ich das Kriterium berechnen?

Ich komme nicht klar mit der ln Funktion.

Danke

Avatar von

1 Antwort

0 Daumen

$$\frac{\frac{1}{ln(k+1)^{k+1}}}{\frac{1}{ln(k)^{k}}}=\frac{ln(k)^{k}}{ln(k+1)^{k+1}}={(\frac{ln(k)}{ln(k+1)})^k}*\frac{1}{ln(k+1)}$$

Der erste Faktor ist eine Potenz mit einer Basis, die kleiner 1 ist, also

ist der ganze Faktor < 1 und du kannst weitermachen mit

$${(\frac{ln(k)}{ln(k+1)})^k}*\frac{1}{ln(k+1)} < \frac{1}{ln(k+1)}$$

und dann gilt  für k>4 sicherlich

$$≤ \frac{1}{ln(5)} < 0,7 $$

Also mit q=0,7 hast du ein q<1 bei dem für k>4 gilt

$$\frac{a_{n+1}}{a_{n}}\leq q \lt 1$$

Die Reihe ist also konvergent.

Avatar von 288 k 🚀

Ein anderes Problem?

Stell deine Frage

Willkommen bei der Mathelounge! Stell deine Frage einfach und kostenlos

x
Made by a lovely community